Difference between revisions of "2013 AMC 10B Problems/Problem 23"

m
(Redirected page to 2013 AMC 12B Problems/Problem 19)
 
(2 intermediate revisions by 2 users not shown)
Line 1: Line 1:
==Problem==
+
#REDIRECT [[2013 AMC 12B Problems/Problem 19]]
In triangle <math>ABC</math>, <math>AB = 13</math>, <math>BC = 14</math>, and <math>CA = 15</math>. Distinct points <math>D</math>, <math>E</math>, and <math>F</math> lie on segments <math>\overline{BC}</math>, <math>\overline{CA}</math>, and <math>\overline{DE}</math>, respectively, such that <math>\overline{AD} \perp \overline{BC}</math>, <math>\overline{DE} \perp \overline{AC}</math>, and <math>\overline{AF} \perp \overline{BF}</math>. The length of segment <math>\overline{DF}</math> can be written as <math>\frac{m}{n}</math>, where <math>m</math> and <math>n</math> are relatively prime positive integers. What is <math>m + n</math>?
 
 
 
 
 
<math>\textbf{(A)}\ 18\qquad\textbf{(B)}\ 21\qquad\textbf{(C)}\ 24\qquad\textbf{(D)}\ 27\qquad\textbf{(E)}\ 30</math>
 
 
 
==Solution==
 

Latest revision as of 12:13, 7 April 2013